0 Daumen
280 Aufrufe

Aufgabe:

Seien \( m \in \mathbb{N} \) und \( M_{m}:=\{0, \ldots, m-1\} \)

i. Geben Sie alle Zahlen \( x \in M_{12} \) an, für die es ein \( y \in M_{12} \) gibt, sodass \( x \cdot y \equiv 1 \) mod 12 gilt.

ii. Seien nun \( m \in \mathbb{N} \) beliebig, \( x \in M_{m} \) und \( \operatorname{gg} \mathrm{T}(m, x)>1 . \) Beweisen Sie, dass es keine Zahl \( y \in M_{m} \) gibt, sodass gilt: \( x \cdot y \equiv 1 \text { mod } m \)



Problem/Ansatz:

kann mir jemand sagen ob ich mit meinem Ergebnis richtig liege?

Es gilt: Mm := {0, ..., m-1},                  x ∈ M12,                     y ∈ M12

Daraus entnehme ich, dass x = 11 und y = 11.

x * y ≡ 1 mod 12

12    ≡ x * y -1, also {1, 122, 243, 364, 485, 606...} , {x * y * n +1 ι  n ∈ ℕ}

x ∈ Mm : {0, 11, 22, 33, 44, 55, 66...}


Beim zweiten Aufgabenteil bin ich mir nicht ganz sicher wie man vorgehen müsste.

Wie könnte man am besten beweisen, dass kein y existiert für welches die Kongruenz gelten würde?


Avatar von

1 Antwort

+1 Daumen



x * y ≡ 1 mod 12

Du hast x=1 und y=1   weil 1*1=1  ≡ 1 mod 12

und denn Fall x=11 und y=11  weil 11*11=121  ≡ 1 mod 12

gefunden. Es gibt auch noch    5*5=25  ≡ 1 mod 12

                                                    7*7=49  ≡ 1 mod 12

Avatar von 288 k 🚀

Ein anderes Problem?

Stell deine Frage

Willkommen bei der Mathelounge! Stell deine Frage einfach und kostenlos

x
Made by a lovely community